Probability generating function (binomial distribution)

Click For Summary
The probability generating function (PGF) for a counting random variable Y is defined as G(s) = E(s^y), applicable for non-negative integers. For a binomial distribution, the PGF can be expressed using the formula G(s) = ∑(n choose y)(p^y)(q^(n-y)), where p is the probability of success and q = 1 - p. The transformation to G(s) = [(1-p) + ps]^n is achieved by recognizing that the sum resembles the binomial expansion. This simplification confirms that the PGF captures the behavior of the binomial distribution effectively. Understanding these steps is crucial for solving related problems in probability theory.
SolidSnake
Messages
17
Reaction score
0

Homework Statement


The probabilty generating funtion G is definied for random varibles whos range are \subset {0,1,2,3,...}. If Y is such a random variable we will call it a counting random varible. Its probabiltiy generating function is G(s) = E(s^{y}) for those s's such that E(|s|^{y})) < \infty.

Homework Equations



binomial distribution = \left(\stackrel{n}{y}\right)p^{y}q^{n-y} , y = 0,1,2,3,...n and 0 \leq p \leq 1

The Attempt at a Solution



What i have so far is...

G(s) = E(s^{y}) = \sum s^{y}\left(\stackrel{n}{y}\right)p^{y}q^{n-y}

G(s) = E(s^{y}) = \sum \left(\stackrel{n}{y}\right)(sp)^{y}q^{n-y}

not sure where to go from that. i managed to do it for the geometric random variable distribution b/c there was no "n choose y". Thanks to wiki, I know what the answer should be. The answer is G(s) = [(1-p) + ps]^{n}. I can't see how they went from what i have above to that.
 
Physics news on Phys.org
well i just realized that G(s) = E(s^{y}) = \sum \left(\stackrel{n}{y}\right)(sp)^{y}q^{n-y}

is the same thing as (q + sp)^{n} .

Also by definition p + q = 1 \Rightarrow q = 1-p which means...

G(s) = E(s^{y}) = [(1-p) + ps]^{n}
 
Last edited:
Question: A clock's minute hand has length 4 and its hour hand has length 3. What is the distance between the tips at the moment when it is increasing most rapidly?(Putnam Exam Question) Answer: Making assumption that both the hands moves at constant angular velocities, the answer is ## \sqrt{7} .## But don't you think this assumption is somewhat doubtful and wrong?

Similar threads

Replies
7
Views
1K
  • · Replies 8 ·
Replies
8
Views
2K
  • · Replies 5 ·
Replies
5
Views
2K
  • · Replies 3 ·
Replies
3
Views
1K
Replies
2
Views
1K
Replies
1
Views
1K
  • · Replies 4 ·
Replies
4
Views
2K
  • · Replies 16 ·
Replies
16
Views
2K
  • · Replies 7 ·
Replies
7
Views
2K
  • · Replies 7 ·
Replies
7
Views
2K